0

A 30 year old lady presented with pulmonary thromboembolism and is found to have APLA. She was given fondaparinux and warfarin and then maintained on warfarin. The appropriate target INR is:
a. 2.0
b. 2.5
c. 3.0
d. 3.5

Isha@2663 Answered question August 16, 2024